Last visit was: 24 Apr 2024, 21:41 It is currently 24 Apr 2024, 21:41

Close
GMAT Club Daily Prep
Thank you for using the timer - this advanced tool can estimate your performance and suggest more practice questions. We have subscribed you to Daily Prep Questions via email.

Customized
for You

we will pick new questions that match your level based on your Timer History

Track
Your Progress

every week, we’ll send you an estimated GMAT score based on your performance

Practice
Pays

we will pick new questions that match your level based on your Timer History
Not interested in getting valuable practice questions and articles delivered to your email? No problem, unsubscribe here.
Close
Request Expert Reply
Confirm Cancel
SORT BY:
Date
Tags:
Difficulty: 505-555 Levelx   Arithmeticx   Rootsx                     
Show Tags
Hide Tags
Senior Manager
Senior Manager
Joined: 11 May 2014
Status:I don't stop when I'm Tired,I stop when I'm done
Posts: 474
Own Kudos [?]: 38821 [217]
Given Kudos: 220
Location: Bangladesh
Concentration: Finance, Leadership
GPA: 2.81
WE:Business Development (Real Estate)
Send PM
Most Helpful Reply
Math Expert
Joined: 02 Sep 2009
Posts: 92900
Own Kudos [?]: 618844 [238]
Given Kudos: 81588
Send PM
GMAT Club Legend
GMAT Club Legend
Joined: 12 Sep 2015
Posts: 6821
Own Kudos [?]: 29917 [15]
Given Kudos: 799
Location: Canada
Send PM
GMAT Tutor
Joined: 24 Jun 2008
Posts: 4128
Own Kudos [?]: 9242 [5]
Given Kudos: 91
 Q51  V47
Send PM
Re: Which of the following is greatest? [#permalink]
4
Kudos
1
Bookmarks
Expert Reply
Hoozan wrote:
From (A) to (E) we see that the integer value keeps falling by 1 unit i.e. 10 - 9 - 8... while the value inside the root keeps increasing by 1 unit i.e. 3 - 4 - 5...

But still, we see that when 10 drops to 9 and when 3 increases to 4 the overall value of the expression increases BUT from C onwards as we continue to follow this pattern (i.e. drop the integer value by 1 unit AND increase the root value by 1) the overall expression keeps falling. Why?? I mean I get the entire calculation in order to solve this question but it was interesting to note that there is a pattern being followed in the answer choices, however, this pattern doesn't help much.

What I also realized is that from (A) to (B) the integer value drops by one i.e. 10 --> 9 while the value of the root increases by 0.3 i.e. √3 --> √4 But from (C) to (E) the value of the root is all most the same i.e. √4 ---> √5 is a 0.2 increase. From √5 to √6 is a 0.2 increase and from √6 to √7 is again a 0.2 increase


Say you extend the list of numbers in the answer choices to go further in both directions:

13√0, 12√1, 11√2, 10√3, ..., 3√10, 2√11, 1√12, 0√13

Notice now that the first number in the list is zero, and the last number is zero. So these numbers can't constantly increase going from left to right, nor can they constantly decrease. They must first increase from zero, then decrease back to zero. If you want to see what this looks like graphically, you can plug the function "y = (13 - x)(√x)" into an online graphing calculator (the answer choices are the points on the graph where x equals the integers from 3 through 7). You'll see you get a curve resembling a downwards parabola which peaks somewhere around x = 4, so it peaks roughly around where our function equals 9√4.

More conceptually, if you take two positive numbers a and b, and find their product ab, and then you decide to add 1 to a, and subtract 1 from b, and find the new product (a+1)(b-1), there's no way to tell if the new product is larger or smaller than the old one, as you can see by first imagining a = 1 and b = 100 (then the new product 2*99 is almost double the old product of 1*100) and then by imagining a = 100 and b = 2 (then the new product 101*1 is roughly half of the old product of 100*2). But instead if I tell you we'll increase a by, say, 20%, and we'll decrease b by, say, 10%, then we can tell if our product will increase: our new product becomes 1.2a * 0.9b = 1.08ab, and our new product is 8% bigger than our old one. So that's what matters: we care about how we're changing each number in percent terms, or in ratio terms, and we don't care about how much we're adding or subtracting. In this question, going from say 4√9 to 5√8, we're increasing '4' by 25% to get to '5', and we're decreasing '√9' by only about 6% to go to '√8', so our product will grow (since 1.25*0.94 is greater than 1). But going from say 9√4 to 10√3, we're now only increasing the '9' by roughly 11%, and we're decreasing the √4 by about 13%, and that will lead to a small decrease.

And for interest only (irrelevant on the GMAT) -- finding precisely where this product (13 - x)(√x) is at a maximum is a calculus problem. In calculus, when we find the "derivative" of a function, we're finding a new function which tells us the slope at every point of our original function. Here we need to use the "product rule" to find the derivative of our function (if you haven't learned calculus, this will all seem very mysterious, but it's something you'll use a lot in an MBA, never on the GMAT though) to find the derivative of y = (13 - x)(√x), and doing that, we learn that the derivative is y' = [ (13 - x) / 2√x ] + (-1)(√x) = [ (13 - x)/2√x ] - √x. When a curve is at its maximum or minimum point, a tangent line to that curve is horizontal, so it has a slope of zero. So if we set the derivative equal to zero, we can locate the maximum point on our original function:

(13 - x)/2√x - √x = 0
(13 - x)/2√x = √x
13 - x = 2x
x = 13/3

So at x = 13/3, our function is at its maximum. Plugging x = 13/3 into our original function y = (13 - x)(√x), we find that function will be at its peak when y = (26/3)(√(13/3)) ~ (8.66)(√4.33). If you compute the numerical value of that product, it's roughly 18.06, so slightly larger than the OA to this question, 9√4 = 18.
General Discussion
Manager
Manager
Joined: 30 Mar 2017
Posts: 53
Own Kudos [?]: 53 [0]
Given Kudos: 14
Send PM
Re: Which of the following is greatest? [#permalink]
A.
Other option have a simultaneous increase in denominator and decrease in denominator.

Sent from my Moto G (5) Plus using GMAT Club Forum mobile app
Board of Directors
Joined: 11 Jun 2011
Status:QA & VA Forum Moderator
Posts: 6072
Own Kudos [?]: 4689 [11]
Given Kudos: 463
Location: India
GPA: 3.5
WE:Business Development (Commercial Banking)
Send PM
Re: Which of the following is greatest? [#permalink]
11
Kudos
AbdurRakib wrote:
Which of the following is greatest?

A. \(10\sqrt{3}\)

B. \(9\sqrt{4}\)

C. \(8\sqrt{5}\)

D. \(7\sqrt{6}\)

E. \(6\sqrt{7}\)


A. 10*1.732 = 17.32
B. 9*2 = 18
C. 8*2.XX = 16.XX
D. 7*2.XX = 14.XX
E. 6*2.XX = 12.XX

Hence, correct answer will be (B)
Intern
Intern
Joined: 13 Apr 2017
Posts: 10
Own Kudos [?]: 20 [0]
Given Kudos: 37
Location: India
Concentration: Finance, Marketing
GRE 1: Q164 V146
WE:Analyst (Computer Software)
Send PM
Re: Which of the following is greatest? [#permalink]
BeingHan wrote:
A.
Other option have a simultaneous increase in denominator and decrease in denominator.

Sent from my Moto G (5) Plus using GMAT Club Forum mobile app



i think you misunderstood the ques.
it is not division., it is square root.
integer* sq.root of an integer
Intern
Intern
Joined: 28 Aug 2016
Posts: 19
Own Kudos [?]: 11 [1]
Given Kudos: 1
Send PM
Re: Which of the following is greatest? [#permalink]
1
Kudos
B. is the answer

A.) square it = 300
B.) square it = 324
C.) square it = 320
D.) square it = 294
E.) square it = 252
Target Test Prep Representative
Joined: 14 Oct 2015
Status:Founder & CEO
Affiliations: Target Test Prep
Posts: 18756
Own Kudos [?]: 22049 [8]
Given Kudos: 283
Location: United States (CA)
Send PM
Re: Which of the following is greatest? [#permalink]
8
Kudos
Expert Reply
AbdurRakib wrote:
Which of the following is greatest?

A. \(10\sqrt{3}\)

B. \(9\sqrt{4}\)

C. \(8\sqrt{5}\)

D. \(7\sqrt{6}\)

E. \(6\sqrt{7}\)


Let’s estimate √3, √5, √6, and √7.

√3 ≈ 1.7

√5 ≈ 2.2

√6 ≈ 2.4

√7 ≈ 2.7

Thus:

A) 10 x 1.7 = 17

B) 9 x 2 = 18

C) 8 x 2.2 = 17.6

D) 7 x 2.4 = 16.8

E) 6 x 2.7 = 16.2

Alternate Solution:

Another way to get rid of the roots to compare each answer is to square each answer choice:

A) (10√3)^2 = 100 x 3 = 300

B) (9√4)^2 = 81 x 4 = 324

C) (8√5)^2 = 64 x 5 = 320

D) (7√6)^2 = 49 x 6 = 294

E) (6√7)^2 = 36 x 7 = 252

Answer: B
Manager
Manager
Joined: 21 Jul 2017
Posts: 154
Own Kudos [?]: 115 [0]
Given Kudos: 143
Location: India
Concentration: Social Entrepreneurship, Leadership
GMAT 1: 660 Q47 V34
GPA: 4
WE:Project Management (Education)
Send PM
Re: Which of the following is greatest? [#permalink]
Bunuel wrote:
Which of the following is greatest?

A. \(10\sqrt{3}=\sqrt{100*3}=\sqrt{300}\)

B. \(9\sqrt{4}=\sqrt{81*4}=\sqrt{324}\)

C. \(8\sqrt{5}=\sqrt{64*5}=\sqrt{320}\)

D. \(7\sqrt{6}=\sqrt{49*6}=\sqrt{294}\)

E. \(6\sqrt{7}=\sqrt{36*7}=\sqrt{252}\)

Answer: B.


HI Bunuel,

I squared all the answer choices and then reached my result. Is that method correct?
Director
Director
Joined: 16 Sep 2016
Status:It always seems impossible until it's done.
Posts: 645
Own Kudos [?]: 2055 [1]
Given Kudos: 174
GMAT 1: 740 Q50 V40
GMAT 2: 770 Q51 V42
Send PM
Re: Which of the following is greatest? [#permalink]
1
Kudos
Yes, it is correct. Since all the numbers are greater than 1, the square of a greater number will be greater.

Best,
Gladi

rever08 wrote:

HI Bunuel,

I squared all the answer choices and then reached my result. Is that method correct?
Tutor
Joined: 16 Oct 2010
Posts: 14817
Own Kudos [?]: 64905 [3]
Given Kudos: 426
Location: Pune, India
Send PM
Re: Which of the following is greatest? [#permalink]
3
Kudos
Expert Reply
Abhishek009 wrote:
AbdurRakib wrote:
Which of the following is greatest?

A. \(10\sqrt{3}\)

B. \(9\sqrt{4}\)

C. \(8\sqrt{5}\)

D. \(7\sqrt{6}\)

E. \(6\sqrt{7}\)


A. 10*1.732 = 17.32
B. 9*2 = 18
C. 8*2.XX = 16.XX
D. 7*2.XX = 14.XX
E. 6*2.XX = 12.XX

Hence, correct answer will be (B)


There is a bit of a problem with this method.

8 * 2.XX needn't necessarily be 16.YY

In fact, 8*2.3 = 18.4
8*2.6 = 20.8
8*2.9 = 23.2
and so on...

So check the method used by Bunuel above.
Manager
Manager
Joined: 13 Nov 2018
Posts: 93
Own Kudos [?]: 111 [0]
Given Kudos: 16
Location: India
GMAT 1: 700 Q51 V32
Send PM
Re: Which of the following is greatest? [#permalink]
AbdurRakib wrote:
Which of the following is greatest?

A. \(10\sqrt{3}\)

B. \(9\sqrt{4}\)

C. \(8\sqrt{5}\)

D. \(7\sqrt{6}\)

E. \(6\sqrt{7}\)



B is correct answer as 9*4^1/2=9*2=18

so OA is B
Director
Director
Joined: 24 Oct 2016
Posts: 583
Own Kudos [?]: 1321 [0]
Given Kudos: 143
GMAT 1: 670 Q46 V36
GMAT 2: 690 Q47 V38
GMAT 3: 690 Q48 V37
GMAT 4: 710 Q49 V38 (Online)
Send PM
Re: Which of the following is greatest? [#permalink]
AbdurRakib wrote:
Which of the following is greatest?

A. \(10\sqrt{3}\)

B. \(9\sqrt{4}\)

C. \(8\sqrt{5}\)

D. \(7\sqrt{6}\)

E. \(6\sqrt{7}\)


Move everything under square root:

A. 100*3 = 300
B. 81*4 = 324
C. 64* 5 = 320
D. 49*6 = 294
E. 36*7 = <300

ANSWER: B
GMAT Club Legend
GMAT Club Legend
Joined: 19 Dec 2014
Status:GMAT Assassin/Co-Founder
Affiliations: EMPOWERgmat
Posts: 21846
Own Kudos [?]: 11665 [1]
Given Kudos: 450
Location: United States (CA)
GMAT 1: 800 Q51 V49
GRE 1: Q170 V170
Send PM
Re: Which of the following is greatest? [#permalink]
1
Kudos
Expert Reply
Hi All,

We’re asked which of the 5 answer choices is GREATEST. Calculating all of those square roots would be tedious; thankfully, we can modify all 5 answer choices in the same way so that we can AVOID doing those specific calculations.

From the wording of the prompt, we know that one of the 5 answers is clearly largest, so if we multiplied all the answers by 2, then the largest answer would still be largest. If we added 1 to each answer, then the largest answer would still be greatest. Etc.

So what can we do to eliminate all of those square root signs? SQUARE each answer…

Answer A becomes (10)(10)(3) = 300
Answer B becomes (9)(9)(4) = 324
Answer C becomes (8)(8)(5) = 320
Answer D becomes (7)(7)(6) = 294
Answer E becomes (6)(6)(7) = 252

Final Answer:

GMAT Assassins aren’t born, they’re made,
Rich
GMAT Club Legend
GMAT Club Legend
Joined: 08 Jul 2010
Status:GMAT/GRE Tutor l Admission Consultant l On-Demand Course creator
Posts: 5957
Own Kudos [?]: 13387 [1]
Given Kudos: 124
Location: India
GMAT: QUANT+DI EXPERT
Schools: IIM (A) ISB '24
GMAT 1: 750 Q51 V41
WE:Education (Education)
Send PM
Re: Which of the following is greatest? [#permalink]
1
Bookmarks
Expert Reply
AbdurRakib wrote:
Which of the following is greatest?

A. \(10\sqrt{3}\)

B. \(9\sqrt{4}\)

C. \(8\sqrt{5}\)

D. \(7\sqrt{6}\)

E. \(6\sqrt{7}\)


Wanna make solving the Official Questions interesting???


Click here and solve 1000+ Official Questions with Video solutions as Timed Sectional Tests
and Dedicated Data Sufficiency (DS) Course


Answer: Option B

Video solution by GMATinsight



Get TOPICWISE: Concept Videos | Practice Qns 100+ | Official Qns 50+ | 100% Video solution CLICK HERE.
Two MUST join YouTube channels : GMATinsight (1000+ FREE Videos) and GMATclub :)
Director
Director
Joined: 28 Sep 2018
Posts: 734
Own Kudos [?]: 559 [1]
Given Kudos: 248
GMAT 1: 660 Q48 V33 (Online)
GMAT 2: 700 Q49 V37
Send PM
Which of the following is greatest? [#permalink]
1
Kudos
IanStewart KarishmaB avigutman BrentGMATPrepNow could you guys share your views on the below thought proces. I want to understand how and why the numbers are functioning in a certain way.


From (A) to (E) we see that the integer value keeps falling by 1 unit i.e. 10 - 9 - 8... while the value inside the root keeps increasing by 1 unit i.e. 3 - 4 - 5...

But still, we see that when 10 drops to 9 and when 3 increases to 4 the overall value of the expression increases BUT from C onwards as we continue to follow this pattern (i.e. drop the integer value by 1 unit AND increase the root value by 1) the overall expression keeps falling. Why?? I mean I get the entire calculation in order to solve this question but it was interesting to note that there is a pattern being followed in the answer choices, however, this pattern doesn't help much.

What I also realized is that from (A) to (B) the integer value drops by one i.e. 10 --> 9 while the value of the root increases by 0.3 i.e. √3 --> √4 But from (C) to (E) the value of the root is all most the same i.e. √4 ---> √5 is a 0.2 increase. From √5 to √6 is a 0.2 increase and from √6 to √7 is again a 0.2 increase
Tutor
Joined: 16 Oct 2010
Posts: 14817
Own Kudos [?]: 64905 [3]
Given Kudos: 426
Location: Pune, India
Send PM
Re: Which of the following is greatest? [#permalink]
2
Kudos
1
Bookmarks
Expert Reply
Hoozan wrote:
IanStewart KarishmaB avigutman BrentGMATPrepNow could you guys share your views on the below thought proces. I want to understand how and why the numbers are functioning in a certain way.


From (A) to (E) we see that the integer value keeps falling by 1 unit i.e. 10 - 9 - 8... while the value inside the root keeps increasing by 1 unit i.e. 3 - 4 - 5...

But still, we see that when 10 drops to 9 and when 3 increases to 4 the overall value of the expression increases BUT from C onwards as we continue to follow this pattern (i.e. drop the integer value by 1 unit AND increase the root value by 1) the overall expression keeps falling. Why?? I mean I get the entire calculation in order to solve this question but it was interesting to note that there is a pattern being followed in the answer choices, however, this pattern doesn't help much.

What I also realized is that from (A) to (B) the integer value drops by one i.e. 10 --> 9 while the value of the root increases by 0.3 i.e. √3 --> √4 But from (C) to (E) the value of the root is all most the same i.e. √4 ---> √5 is a 0.2 increase. From √5 to √6 is a 0.2 increase and from √6 to √7 is again a 0.2 increase


Hoozan

The decrease is of 1 each from 10 to 9 to 8 etc but from root 3 to root 4 to root 5 etc, the numbers are increasing by different amounts. Hence, it doesn't follow a defined increasing or decreasing pattern. Also note that even if the increase/decrease is by the same amount, the relation is a quadratic.

Say we compare

100*2
95*3
90*4
... and so on

f(x) = (100 - 5a)(2 + a)
a goes from 0 to 1 to 2 etc...

This is a quadratic. We know that a quadratic is a parabola. So whether the values will be increasing or decreasing or will reach a maximum and then decrease or reach a minimum and then increase depends on what part of the parabola you are dealing with.

But yes, keep thinking of patterns and numbers! Good job!
Manager
Manager
Joined: 12 Oct 2015
Posts: 238
Own Kudos [?]: 360 [0]
Given Kudos: 144
Location: Canada
Concentration: Leadership, Accounting
GMAT 1: 700 Q47 V39
GPA: 3
WE:Accounting (Accounting)
Send PM
Re: Which of the following is greatest? [#permalink]
The simplest approach for me is to square everything.

A) 100*3 = 300
B) 81*4 = 324
C) 64*5 = 320
D) 49*6 = <300
E) 36*7 = <300

B is correct.
Director
Director
Joined: 28 Sep 2018
Posts: 734
Own Kudos [?]: 559 [0]
Given Kudos: 248
GMAT 1: 660 Q48 V33 (Online)
GMAT 2: 700 Q49 V37
Send PM
Re: Which of the following is greatest? [#permalink]
IanStewart wrote:
Hoozan wrote:
From (A) to (E) we see that the integer value keeps falling by 1 unit i.e. 10 - 9 - 8... while the value inside the root keeps increasing by 1 unit i.e. 3 - 4 - 5...

But still, we see that when 10 drops to 9 and when 3 increases to 4 the overall value of the expression increases BUT from C onwards as we continue to follow this pattern (i.e. drop the integer value by 1 unit AND increase the root value by 1) the overall expression keeps falling. Why?? I mean I get the entire calculation in order to solve this question but it was interesting to note that there is a pattern being followed in the answer choices, however, this pattern doesn't help much.

What I also realized is that from (A) to (B) the integer value drops by one i.e. 10 --> 9 while the value of the root increases by 0.3 i.e. √3 --> √4 But from (C) to (E) the value of the root is all most the same i.e. √4 ---> √5 is a 0.2 increase. From √5 to √6 is a 0.2 increase and from √6 to √7 is again a 0.2 increase


Say you extend the list of numbers in the answer choices to go further in both directions:

13√0, 12√1, 11√2, 10√3, ..., 3√10, 2√11, 1√12, 0√13

Notice now that the first number in the list is zero, and the last number is zero. So these numbers can't constantly increase going from left to right, nor can they constantly decrease. They must first increase from zero, then decrease back to zero. If you want to see what this looks like graphically, you can plug the function "y = (13 - x)(√x)" into an online graphing calculator (the answer choices are the points on the graph where x equals the integers from 3 through 7). You'll see you get a curve resembling a downwards parabola which peaks somewhere around x = 4, so it peaks roughly around where our function equals 9√4.

More conceptually, if you take two positive numbers a and b, and find their product ab, and then you decide to add 1 to a, and subtract 1 from b, and find the new product (a+1)(b-1), there's no way to tell if the new product is larger or smaller than the old one, as you can see by first imagining a = 1 and b = 100 (then the new product 2*99 is almost double the old product of 1*100) and then by imagining a = 100 and b = 2 (then the new product 101*1 is roughly half of the old product of 100*2). But instead if I tell you we'll increase a by, say, 20%, and we'll decrease b by, say, 10%, then we can tell if our product will increase: our new product becomes 1.2a * 0.9b = 1.08ab, and our new product is 8% bigger than our old one. So that's what matters: we care about how we're changing each number in percent terms, or in ratio terms, and we don't care about how much we're adding or subtracting. In this question, going from say 4√9 to 5√8, we're increasing '4' by 25% to get to '5', and we're decreasing '√9' by only about 6% to go to '√8', so our product will grow (since 1.25*0.94 is greater than 1). But going from say 9√4 to 10√3, we're now only increasing the '9' by roughly 11%, and we're decreasing the √4 by about 13%, and that will lead to a small decrease.

And for interest only (irrelevant on the GMAT) -- finding precisely where this product (13 - x)(√x) is at a maximum is a calculus problem. In calculus, when we find the "derivative" of a function, we're finding a new function which tells us the slope at every point of our original function. Here we need to use the "product rule" to find the derivative of our function (if you haven't learned calculus, this will all seem very mysterious, but it's something you'll use a lot in an MBA, never on the GMAT though) to find the derivative of y = (13 - x)(√x), and doing that, we learn that the derivative is y' = [ (13 - x) / 2√x ] + (-1)(√x) = [ (13 - x)/2√x ] - √x. When a curve is at its maximum or minimum point, a tangent line to that curve is horizontal, so it has a slope of zero. So if we set the derivative equal to zero, we can locate the maximum point on our original function:

(13 - x)/2√x - √x = 0
(13 - x)/2√x = √x
13 - x = 2x
x = 13/3

So at x = 13/3, our function is at its maximum. Plugging x = 13/3 into our original function y = (13 - x)(√x), we find that function will be at its peak when y = (26/3)(√(13/3)) ~ (8.66)(√4.33). If you compute the numerical value of that product, it's roughly 18.06, so slightly larger than the OA to this question, 9√4 = 18.


Thank you so much for this :) Was a great read and a super cool conceptual understanding
GMAT Club Bot
Re: Which of the following is greatest? [#permalink]
 1   2   
Moderators:
Math Expert
92900 posts
Senior Moderator - Masters Forum
3137 posts

Powered by phpBB © phpBB Group | Emoji artwork provided by EmojiOne